OFO 2023 Problema 9

Problemas que aparecen en el Archivo de Enunciados.
Avatar de Usuario
Sandy

OFO - Medalla de Bronce-OFO 2019 OFO - Medalla de Plata-OFO 2020 FOFO Pascua 2020 - Copa-FOFO Pascua 2020 COFFEE - Mención-COFFEE Ariel Zylber FOFO 10 años - Copa-FOFO 10 años
OFO - Medalla de Oro-OFO 2021 FOFO 11 años - Medalla-FOFO 11 años OFO - Medalla de Plata-OFO 2022 OFO - Jurado-OFO 2023 FOFO 13 años - Jurado-FOFO 13 años
OFO - Jurado-OFO 2024
Mensajes: 280
Registrado: Lun 27 Nov, 2017 1:59 am
Medallas: 11
Nivel: 3

OFO 2023 Problema 9

Mensaje sin leer por Sandy »

Sea $\mathbb{N}$ el conjunto de los enteros positivos. Determinar todas las funciones $f:\mathbb{N}\to \mathbb{N}$ tales que para todos $a,b$ enteros positivos, hay dos de $f(a)$, $f(b)$, $f(a+b)$ que suman el tercero.
Fallo inapelable.
Avatar de Usuario
Sandy

OFO - Medalla de Bronce-OFO 2019 OFO - Medalla de Plata-OFO 2020 FOFO Pascua 2020 - Copa-FOFO Pascua 2020 COFFEE - Mención-COFFEE Ariel Zylber FOFO 10 años - Copa-FOFO 10 años
OFO - Medalla de Oro-OFO 2021 FOFO 11 años - Medalla-FOFO 11 años OFO - Medalla de Plata-OFO 2022 OFO - Jurado-OFO 2023 FOFO 13 años - Jurado-FOFO 13 años
OFO - Jurado-OFO 2024
Mensajes: 280
Registrado: Lun 27 Nov, 2017 1:59 am
Medallas: 11
Nivel: 3

Re: OFO 2023 Problema 9

Mensaje sin leer por Sandy »

Solución:
Spoiler: mostrar
Vamos a decirle $P(a,b)$ a la condición del enunciado.
Veremos por inducción fuerte que $f(n)=n*f(1)$ para todo $n$. Primero, unos casos base:
$P(a,a)$ nos da que $f(2a)=2*f(a)$ o que $f(a)=f(2a)+f(a)$. Como el segundo caso nos da $f(2a)=0$, concluimos que $f(2a)=2*f(a)$.
Luego $f(2)=2*f(1)$ y $f(4)=2*f(2)=2*2*f(1)=4*f(1)$.
$P(1,2)$ nos da que $f(3)=f(1)+f(2)=3*f(1)$ o que $f(3)=f(2)-f(1)=f(1)$.
$P(1,3)$ nos da que $f(3)=f(1)+f(4)=5*f(1)$ o que $f(3)=f(4)-f(1)=3*f(1)$.
Como $f(1)\neq 0$, debe ser que $f(3)=3*f(1)$, al ser la única opción en ambas listas de posibilidades.

Ahora, sea $n\geq 5$.Supongamos que para $k<n$ vale que $f(k)=k*f(1)$.
$P(1,n-1)$ nos da que $f(n)=f(1)+f(n-1)=n*f(1)$ o que $f(n)=f(n-1)-f(1)=(n-2)*f(1)$.
$P(2,n-2)$ nos da que $f(n)=f(2)+f(n-2)=n*f(1)$ o que $f(n)=f(n-2)-f(2)=(n-4)*f(1)$.
Pero de nuevo, como $f(1)\neq 0$, debe ser que $f(n)=n*f(1)$, al ser la única opción en ambas listas de posibilidades. Esto concluye la inducción.

Sólo nos queda chequear que todas las funciones de la forma $f(n)=nk$, con $k>0$, cumplen.
$f(a)+f(b)=ak+bk=(a+b)k=f(a+b)$, por lo que se cumple la condición del enunciado.

Luego las únicas funciones que satisfacen son las de la forma $f(n)=nk$, donde $k$ es un entero positivo fijo.
Última edición por Sandy el Mar 07 Feb, 2023 1:04 am, editado 2 veces en total.
Fallo inapelable.
Avatar de Usuario
Tiziano Brunelli

OFO - Medalla de Plata-OFO 2023 FOFO 13 años - Medalla-FOFO 13 años OFO - Mención-OFO 2024
Mensajes: 99
Registrado: Dom 21 Ago, 2022 1:24 pm
Medallas: 3
Nivel: Exolímpico
Ubicación: Al lado de Alta Córdoba, Córdoba capital, Córdoba

Re: OFO 2023 Problema 9

Mensaje sin leer por Tiziano Brunelli »

Sandy escribió: Vie 27 Ene, 2023 12:03 am Sea $\mathbb{N}$ el conjunto de los enteros positivos. Determinar todas las funciones $f:\mathbb{N}\to \mathbb{N}$ tales que para todos $a,b$ enteros positivos, hay dos de $f(a)$, $f(b)$, $f(a+b)$ que suman el tercero.
Spoiler: mostrar
Spoiler: mostrar
al final de la biblia muere jesús :o
Spoiler: mostrar
Supongamos que la función evaluada en 1 da $k \in \mathbb{N}$, entonces al ser $2=1+1$ se tiene o que $f(2)=f(1)+f(1)=2k$ o que $f(1)=f(2)+f(1) \rightarrow f(2)=0$, la segunda no puede ser porque 0 no es parte de los enteros postivos, por lo que $f(2)=2k$. Ahora supongamos que existe un $m\geq 2 \in \mathbb{N} ∣ \forall i\leq m:$
$f(i)=ik$
tenemos entonces que $f(m+1)=f(m)+f(1)$ ó $f(m+1)+f(m)=f(1)$ ó $f(m+1)+f(1)=f(m)$, lo que nos da $f(m+1)=(m+1)k; f(m+1)= k(1-m) ó f(m+1)=k(m-1)$ respectivamente, la opción del medio no puede ser porque al ser $k\geq 1$ entonces $f(m+1)$ no estaría en los enteros positivos,y la tercera tampoco porque también o $f(m+1)=f(m-1)+f(2) \rightarrow f(m+1)=(k+1)m$ ó $f(m+1)+f(m-1)=f(2) \rightarrow f(m+1)=k(2-m)$ (no es posible porque de nuevo haría que f(m+1) esté fuera de los enteros postiivos) ó $f(m+1)+f(2)=f(m-1) \rightarrow f(m+1)=k(m-2)$ y obviamente $k(m+1)\neq k(m-1)\neq k(m-2)$, por lo que $f(m+1)\neq (m-1)k$, por ende $f(m+1)=(m+1)k$. Por inducción y siendo el caso base m=2 se obtiene que para todo $x \in \mathbb{N}: f(x)=kx$ es la única solución con $k \in \mathbb{N}$, y obviamente si $k \notin \mathbb{N}$ entonces habrán resultados por fuera de los enteros positivos. Para testear que efectivamente esto pasa tengamos que: $f(a+b)=f(a)+f(b) \rightarrow (a+b)k=ak+bk$ lo cual es efectivamente cierto por la propiedad distributiva, nótese que solo se cumplirá $f(a+b)=f(a)+f(b)$ por la hipótesis inductiva que mostré y demostré.
RTA:Las únicas soluciones son las funciones de la forma $f(a)=ak$; con $k \in \mathbb{N}$
"cada vez que uses xor, piensa en mí, estaré usando vectores módulo 2"- un cordobés a otro. :D
Avatar de Usuario
fran :)

FOFO 12 años - Mención-FOFO 12 años OFO - Medalla de Plata-OFO 2023 FOFO 13 años - Copa-FOFO 13 años
Mensajes: 21
Registrado: Mié 22 Jun, 2022 6:04 pm
Medallas: 3
Nivel: 3
Ubicación: Buenos Aires

Re: OFO 2023 Problema 9

Mensaje sin leer por fran :) »

Spoiler: mostrar
Parece que mi solución quedó muy complicada
Spoiler: mostrar
Para empezar, la condición que cumple $f$ es equivalente a decir que para todo $a, b$ una de estas tres se cumple:
  • $f(a) + f(b) = f(a + b)$
  • $f(a + b) + f(b) = f(a)$
  • $f(a) + f(a + b) = f(b)$
Esto lo puedo reescribir como:
  • $f(a + b) = f(a) + f(b)$
  • $f(a + b) = f(a) - f(b)$
  • $f(a + b) = - f(a) + f(b)$
Primero voy a probar por inducción que para toda potencia de dos $2^k$, $f(2^k) = 2^kf(1)$
Spoiler: mostrar
Caso base: $k = 0$. $2^0 = 1$, entonces $f(2^k) = f(1) = 1 \cdot f(1) = 2^kf(1)$, por lo cual esto es verdadero.
Paso inductivo: $k = i + 1$.
Hipótesis inductiva: $f(2^i) = 2^if(1)$
Consideremos $f(2^{i+1})$. Esto es igual a $f(2^i+2^i)$
Por la condicion de f, una de estas dos se cumple:
  • $f(2^i+2^i) = f(2^i)+f(2^i)$
  • $f(2^i) = f(2^i+2^i)+f(2^i)$
El segundo caso es falso, porque si restamos $f(2^i)$ de los dos lados nos queda que $f(2^i+2^i) = 0$, que es falso porque $0$ no esta en el codominio de $f$ que es $\mathbb{N}$

Entonces el primer caso es verdadero. $f(2^i) = 2^if(1)$ por la hipotesis inductiva entonces:

$f(2^i+2^i) = f(2^i)+f(2^i)$
$f(2^i+2^i) = 2^if(1)+2^if(1)$
$f(2^{i+1}) = 2^{i+1}f(1)$
$f(2^k) = 2^k f(1)$
Por lo que demostramos lo que queriamos probar.
Ahora pruebo por induccion que para todo $x$, $f(x) = xf(1)$.
Spoiler: mostrar
Caso base: x = 1. $f(x) = f(1) = 1 \cdot f(1) = xf(1)$. QED.

Paso inductivo:
Spoiler: mostrar
$x = i + 1$
Hipótesis inductiva: $f(i) = if(1)$

Consideremos $f(x) = f(i + 1)$. Por la propiedad que cumple $f$, una de estas tres se cumple:
  • $f(i + 1) = f(i) + f(1)$
  • $f(i + 1) = f(i) - f(1)$
  • $f(i + 1) = - f(i) + f(1)$
Recordemos que por la hipotesis inductive $f(i) = if(1)$. Esto se cumple para todo $i$ menor a $x$.

El tercer caso es falso:
Spoiler: mostrar
$f(1) = f(i) + f(i + 1)$
$f(1) = if(1) + f(i + 1)$
$f(1) - if(1) = f(i + 1)$
$(1-i)f(1) = f(i + 1)$

$i \geq 1$ porque estamos haciendo induccion con la base de $x = 1$. Entonces $(1-i) \leq 0$
$f(1) > 0$ porque el codominio de $f$ son los naturales
Entonces $(1-i)f(1) \leq 0$, pero $f(i + 1) > 0$ porque el codominio de $f$ son los naturales. Entonces llegamos a una contradiccion.
El segundo caso es falso:
Spoiler: mostrar
Esto es mas complicado.

$f(i + 1) = f(i) - f(1)$
$f(i + 1) = if(1) - f(1)$
$f(i + 1) = (i - 1)f(1)$

Esta conclusión no es immediatamente contradictoria.

Pero ahora voy a probar (esta vez sin inducción) que esto implica para todo $2i \geq m > i + 1$, $f(m) = f(2i - m)f(1)$
Spoiler: mostrar
Sea $m = k + 1$
$2i > k > i$

$k < 2i \to k - i < i \to k - i + 1 < i + 1 \to (k - i + 1) < i + 1$, entones la hipótesis inductiva de antes cumple para $(k - i + 1)$.
$f(k - i + 1) = (k - i + 1)f(1)$

Una de estas tres se cumple
  • $f(k + 1) = f(i + (k - i + 1)) = f(i) + f(k - i + 1) = if(1) + (k - i + 1)f(1) = (k + 1)f(1)$
  • $f(k + 1) = f(i + (k - i + 1)) = f(i) - f(k - i + 1) = if(1) - (k - i + 1)f(1) = (2i - k - 1)f(1)$
  • $f(k + 1) = f(i + (k - i + 1)) = - f(i) + f(k - i + 1) = - if(1) + (k - i + 1)f(1) = (-2i + k + 1)f(1)$. Esto no puede ser porque como $m \leq 2i$, $(-2i + k + 1) \leq 0$.
Pero también una de estas tres se cumple
  • $f(k + 1) = f(i + 1 + (k - i)) = f(i + 1) + f(k - i) = (i - 1)f(1) + (k - i)f(1) = (k - 1)f(1)$
  • $f(k + 1) = f(i + 1 + (k - i)) = f(i + 1) - f(k - i) = (i - 1)f(1) - (k - i)f(1) = (2i - k - 1)f(1)$
  • $f(k + 1) = f(i + 1 + (k - i)) = - f(i + 1) + f(k - i) = - (i - 1)f(1) + (k - i)f(1) = (-2i + k + 1)f(1)$ Esto no puede ser porque como $m \leq 2i$, $(-2i + k + 1) \leq 0$.
Observemos que el valor que elijamos de arriba tiene que ser igual al de abajo por la transitividad de la igualdad.

$k - 1 \neq 2i - k - 1$
Spoiler: mostrar
$k - 1 \neq 2i - k - 1$
$k \neq 2i - k$
$2k \neq 2i$
$k \neq i$
verdadero porque $m > i + 1$
$k + 1 \neq 2i - k - 1$
Spoiler: mostrar
$k +1 1 \neq 2i - k - 1$
$k + 2 \neq 2i - k$
$k + 2 \neq 2i - k$
$2k + 2 \neq 2i$
$k + 1 \neq i$
$m \neq i$
verdadero porque $m > i$
Como $f(i + 1 + (k - i))$ = $f(i + (k - i + 1))$, la única opción es que $f(k + 1) = (2i - k - 1)f(1) \to f(m) = (2i - m)f(1)$

Entonces, $f(k + 1) = (2i - k - 1)f(1)$
Sabiendo esto, hay dos maneras distintas que encontré de llegar a una contradicción:
Spoiler: mostrar

Para esto voy a usar una propiedad que tienen las potencias de dos, que es que para todo $x$ natural (no potencia de dos) existe un $2^k > x$ tal que $x > \frac{2^k}{2}$. Por lo tanto, para todo $i$ existe una potencia de 2 tal que $i < 2^k \leq 2i$. La pruba es bastante obvia, pero es esta:
Spoiler: mostrar
Consideremos la menor potencia de dos $2^k$ mayor a $x$.

Si no se cumple que $x > \frac{2^k}{2}$, entonces como $\frac{2^k}{2} = 2^{k-1}$ es mayor a $x$, entonces $2^{k-1}$ es una potencia de 2 mayor a $x$ pero menor a $2^k$, que contradice lo que asumimos al principio que es que $2^k$ es la menor potencia de dos mayor a $x$. Entonces llegamos a un absurdo.

Por lo tanto $x > \frac{2^k}{2}$.
Para $m = 2^p$ (siempre existe una potencia de dos entre $i$ y $2i$ inclusive)
$f(m) = f(2^p)$
$(-2i + m)f(1) = 2^pf(1)$
$2^p = -2i + m$
$m = -2i + m$
$-2i = 0$, que es falso.
Spoiler: mostrar
Para $m = 2i$:
$f(m)= f(2i) = (-2i +m)f(1) = 0f(1) = 0$, pero esto no es posible (!).
Entonces llegamos a una contradicción
Entonces el primer caso es verdadero.

$f(i + 1) = f(i) + f(1)$
$f(i + 1) = if(1) + f(1)$
$f(i + 1) = (i + 1)f(1)$
$f(x) = xf(1)$.
Entonces $f(x) = xf(1)$. Como $f(1)$ es constante significa que esta función es lineal.

Es fácil comprobar que para todo $k$, $f(x) = kx$ cumple con la condición:

$f(a + b) = f(a) + f(b)$
$k(a + b) = ka + kb$
$ka + kb = ka + kb$

Entonces la respuesta es que para todo $k$, $f(x) = kx$. Esos son los únicos valores posibles para $f$.
$$\phantom{[muajajacaracteresinfinitos=}\begin{matrix}(λx.F\;a)&→&x=a;F\\(\{x_0\;x_1\}\;a)&→&\{a_0\;a_1\}=a;\{(x_0\;a_0)\;(x_1\;a_1)\}\\\{a_0\;a_1\}=\{b_0\;b_1\}&→&a_0=b_0;a_1=b_1\\\{a_0\;a_1\}=λx.F&→&x=\{x_0\;x_1\};\{f_0\;f_1\}=F;a_0=λx_0.f_0;a_1=λx_1.f_1\end{matrix}\phantom{]}$$
([{&}])
Mensajes: 18
Registrado: Sab 10 Jun, 2023 4:38 pm
Nivel: 2

Re: OFO 2023 Problema 9

Mensaje sin leer por ([{&}]) »

Sandy escribió: Dom 05 Feb, 2023 9:19 pm Solución:
Spoiler: mostrar
Vamos a decirle $P(a,b)$ a la condición del enunciado.
Veremos por inducción fuerte que $f(n)=n*f(1)$ para todo $n$. Primero, unos casos base:
$P(a,a)$ nos da que $f(2a)=2*f(a)$ o que $f(a)=f(2a)+f(a)$. Como el segundo caso nos da $f(2a)=0$, concluimos que $f(2a)=2*f(a)$.
Luego $f(2)=2*f(1)$ y $f(4)=2*f(2)=2*2*f(1)=4*f(1)$.
$P(1,2)$ nos da que $f(3)=f(1)+f(2)=3*f(1)$ o que $f(3)=f(2)-f(1)=f(1)$.
$P(1,3)$ nos da que $f(3)=f(1)+f(4)=5*f(1)$ o que $f(3)=f(4)-f(1)=3*f(1)$.
Como $f(1)\neq 0$, debe ser que $f(3)=3*f(1)$, al ser la única opción en ambas listas de posibilidades.

Ahora, sea $n\geq 5$.Supongamos que para $k<n$ vale que $f(k)=k*f(1)$.
$P(1,n-1)$ nos da que $f(n)=f(1)+f(n-1)=n*f(1)$ o que $f(n)=f(n-1)-f(1)=(n-2)*f(1)$.
$P(2,n-2)$ nos da que $f(n)=f(2)+f(n-2)=n*f(1)$ o que $f(n)=f(n-2)-f(2)=(n-4)*f(1)$.
Pero de nuevo, como $f(1)\neq 0$, debe ser que $f(n)=n*f(1)$, al ser la única opción en ambas listas de posibilidades. Esto concluye la inducción.

Sólo nos queda chequear que todas las funciones de la forma $f(n)=nk$, con $k>0$, cumplen.
$f(a)+f(b)=ak+bk=(a+b)k=f(a+b)$, por lo que se cumple la condición del enunciado.

Luego las únicas funciones que satisfacen son las de la forma $f(n)=nk$, donde $k$ es un entero positivo fijo.
Pregunta:
al determinar al principio que si se cumplían dos de las tres situaciones entonces f(2a) =0. Eso no descarta por completo esas situaciones?
porque en el enunciado dice que las funciones debian tener codominio natural y 0 no es natural.
Responder